# Ch. XYZ: April 6 ## Multiplicativity in Towers :::{.remark} An important topic not in the book: relative extensions of number fields, vs absolute extensions over $\QQ$. \begin{tikzcd} L && {\ZZ_?} && Q \\ \\ K && {\ZZ_K} && {P = Q \intersect \ZZ_K} \arrow[no head, from=1-5, to=3-5] \arrow[no head, from=1-3, to=3-3] \arrow[no head, from=1-1, to=3-1] \end{tikzcd} > [Link to Diagram](https://q.uiver.app/?q=WzAsNixbMCwwLCJMIl0sWzAsMiwiSyJdLFsyLDAsIlxcWlpfPyJdLFsyLDIsIlxcWlpfSyJdLFs0LDAsIlEiXSxbNCwyLCJQID0gUSBcXGludGVyc2VjdCBcXFpaX0siXSxbNCw1LCIiLDAseyJzdHlsZSI6eyJoZWFkIjp7Im5hbWUiOiJub25lIn19fV0sWzIsMywiIiwwLHsic3R5bGUiOnsiaGVhZCI6eyJuYW1lIjoibm9uZSJ9fX1dLFswLDEsIiIsMCx7InN0eWxlIjp7ImhlYWQiOnsibmFtZSI6Im5vbmUifX19XV0=) Note that if $Q\contains P$, then $Q$ will divide the extension of $P$ to $\ZZ_L$, if $Q$ will show up in the prime factorization of $P\ZZ_L$. We defined $e(Q/P)$ to be the exponent of $Q$ in the factorization of $P\ZZ_L$, and $f(Q/P)$ to be the degree of the field extension $\ZZ_L/Q$ over $\ZZ_K/P$. ::: :::{.remark} Note that "lying above" is transitive, and the following situation makes sense: \begin{tikzcd} M && {P''} \\ \\ L && {P'} \\ \\ K && P \arrow[no head, from=3-1, to=5-1] \arrow[no head, from=1-1, to=3-1] \arrow[no head, from=1-3, to=3-3] \arrow[no head, from=3-3, to=5-3] \end{tikzcd} > [Link to Diagram](https://q.uiver.app/?q=WzAsNixbMCwyLCJMIl0sWzAsNCwiSyJdLFswLDAsIk0iXSxbMiw0LCJQIl0sWzIsMiwiUCciXSxbMiwwLCJQJyciXSxbMCwxLCIiLDAseyJzdHlsZSI6eyJoZWFkIjp7Im5hbWUiOiJub25lIn19fV0sWzIsMCwiIiwwLHsic3R5bGUiOnsiaGVhZCI6eyJuYW1lIjoibm9uZSJ9fX1dLFs1LDQsIiIsMCx7InN0eWxlIjp7ImhlYWQiOnsibmFtZSI6Im5vbmUifX19XSxbNCwzLCIiLDAseyJzdHlsZSI6eyJoZWFkIjp7Im5hbWUiOiJub25lIn19fV1d) How are the various $e$ and $f$ related? It turns out they work similarly to degrees of field extensions: ::: :::{.theorem title="Multiplicativity in towers"} \[ e(P''/P) &= e(P''/P') \cdot e(P'/P) \\ f(P''/P) &= f(P''/P') \cdot f(P'/P) .\] ::: :::{.proof title="?"} Note that $f$ is the degree of a field extension where we take $\ZZ_K/P \embeds \ZZ_L$ and consider the induced quotient map to $\ZZ_L/Q$. By composing inclusions we have a commutative diagram: \begin{tikzcd} {\ZZ_K/P} && {\ZZ_L/P'} && {\ZZ_M/P''} \arrow[hook, from=1-1, to=1-3] \arrow[hook, from=1-3, to=1-5] \arrow[curve={height=30pt}, hook, from=1-1, to=1-5] \end{tikzcd} > [Link to Diagram](https://q.uiver.app/?q=WzAsMyxbMCwwLCJcXFpaX0svUCJdLFsyLDAsIlxcWlpfTC9QJyJdLFs0LDAsIlxcWlpfTS9QJyciXSxbMCwxLCIiLDAseyJzdHlsZSI6eyJ0YWlsIjp7Im5hbWUiOiJob29rIiwic2lkZSI6InRvcCJ9fX1dLFsxLDIsIiIsMCx7InN0eWxlIjp7InRhaWwiOnsibmFtZSI6Imhvb2siLCJzaWRlIjoidG9wIn19fV0sWzAsMiwiIiwyLHsiY3VydmUiOjUsInN0eWxlIjp7InRhaWwiOnsibmFtZSI6Imhvb2siLCJzaWRlIjoidG9wIn19fV1d) So for $f$, this reduces to the multiplicativity of degrees in towers of field extensions. For $e$, write $P\ZZ_L = P'^{e(P'/P)} I$ where $I\in \Id(\ZZ_L)$ is nonzero and $P'\notdivides I$. Similarly we can write $P' \ZZ_< = P''^{e(P'/P')} J$ where $J\in \Id(\ZZ_L)$ and $P''\notdivides J$. Using general facts about ideal extensions and substituting yields \[ P\ZZ_M &= (P\ZZ_L)\ZZ_M \\ &= (P'\ZZ_M)^{e(P'/P)} I\ZZ_M \\ &= P''^{e(P'/P) e(P'/P')} J^{e(P'/P)} (I\ZZ_M) \\ .\] We know that $P''\notdivides J$, so it doesn't divide any power of $J$ either. We can check that $P''\notdivides I\ZZ_M$: otherwise, if it did then $P''\supseteq I\ZZ_M$ and \[ P'' \intersect \ZZ_L \supseteq (I\ZZ_M) \intersect \ZZ_L \supset I \] by intersecting both sides with $\ZZ_L$. But we know $P'' \intersect \ZZ_L = P'$ since $P''$ was above $P'$, and $P'\notdivides I$ and thus $P'\not\contains I$. ::: :::{.definition title="Splitting completely"} Suppose $L/K$ is an extension of number fields and say $P\supseteq \ZZ_K$ is a nonzero prime ideal. Then $P$ **splits completely** in $L$ (or $\ZZ_L$) if $e(G/P) = f(G/P) = 1$ for all $G$ above $P$ in $\ZZ_L$. Equivalently, there is a factorization $P\ZZ_L = \prod_{i\leq n} Q_i$ with the $Q_i$ distinct and $n \da [L: K]$. ::: :::{.proposition title="?"} Let $M/L/K$ be a tower of number fields and $P\in \spec \ZZ_K$. Then if $P$ splits completely in $M$, it splits completely in $L$. ::: :::{.slogan} Splitting completely in an extension implies splitting completely in every intermediate extension. ::: :::{.proof title="?"} Suppose $P$ splits completely in $M$, we then want to show that for any $P'$ in $\ZZ_L$ above $P$, then $e=f=1$. We know $P'$ has some prime factor, so choose any $P''$ in $\ZZ_M$ above $P'$: \begin{tikzcd} M && {\exists P''} \\ \\ L && {P'} \\ \\ K && P \arrow[no head, from=1-1, to=3-1] \arrow[no head, from=3-1, to=5-1] \arrow[no head, from=5-3, to=3-3] \arrow[no head, from=1-3, to=3-3] \end{tikzcd} > [Link to Diagram](https://q.uiver.app/?q=WzAsNixbMCw0LCJLIl0sWzAsMiwiTCJdLFswLDAsIk0iXSxbMiw0LCJQIl0sWzIsMiwiUCciXSxbMiwwLCJcXGV4aXN0cyBQJyciXSxbMiwxLCIiLDAseyJzdHlsZSI6eyJoZWFkIjp7Im5hbWUiOiJub25lIn19fV0sWzEsMCwiIiwwLHsic3R5bGUiOnsiaGVhZCI6eyJuYW1lIjoibm9uZSJ9fX1dLFszLDQsIiIsMCx7InN0eWxlIjp7ImhlYWQiOnsibmFtZSI6Im5vbmUifX19XSxbNSw0LCIiLDIseyJzdHlsZSI6eyJoZWFkIjp7Im5hbWUiOiJub25lIn19fV1d) Since $P''$ is above $P$ and $e(P''/P) = f(P''/P) =1$, we can use multiplicativity in towers: \[ e(P'/P) \divides e(P''/P) = 1 &\implies e(P'/P) = 1 \\ f(P'/P) \divides f(P''/P) = 1 &\implies f(P'/P) = 1 \\ .\] ::: :::{.definition title="?"} If $L/K$ is an extension of number fields and $P\in \spec \ZZ_K$ is nonzero, then $P$ **ramifies** in $L$ (or $\ZZ_L$) if $e(Q/P)>1$ for some $Q\in \spec \ZZ_L$ above $P$. ::: :::{.proposition title="?"} For $M/L/K$ a tower of number fields and $P\in \spec \ZZ_K$ is nonzero and unramified in $M$, then $P$ is unramified in $L$. ::: :::{.proof title="?"} Same as the last proof. ::: ## Galois Theory and Prime Decomposition :::{.remark} This will lead into defining the Frobenius element and the fundamental theorem of algebraic number theory: Chebotarev Density. Some setup/notation: - $L/K$ will be a Galois extension of number of fields - $P \in \spec \ZZ_K, Q\in \spec \ZZ_L$ will be nonzero prime ideals. We may or may not require $Q$ to lie above $P$. ::: :::{.remark} \envlist - Any $\sigma \in G(L/K)$ is an automorphism of $L$ fixing $K$, which restricts to an automorphism of $\ZZ_L$ since it preserves algebraic integers. - It also preserves prime ideals. - Suppose $Q$ lies above $P$, then $P\ZZ_L = Q^{e(Q/p)} \cdots$. $\sigma$ fixes $K$ pointwise, so applying it to both sides yields $P\ZZ_L = \sigma(Q)^{e(Q/P)}\cdots$ where $\sigma(Q)$ shows up with the exact same power since it preserves distinctness of ideals. - In particular, $\sigma(G)$ lies above $P$, so $G(L/K)$ acts on the set of primes above $P$, and it turns out (very importantly) to be transitive. ::: :::{.theorem title="?"} The action of $G(L/K)$ on primes above $P$ is transitive. ::: :::{.proof title="?"} Let $Q_1, Q_2$ be primes above $P$ and suppose toward a contradiction that $\Orb(Q_1)$ does not contain $Q_2$. We may choose $\alpha\in \ZZ_L$ such that - $\alpha \equiv 0 \mod Q_2$ - $\alpha \equiv 1 \mod Q$ for all $Q$ of the form $Q = \sigma(Q_1)$, noting that none are $Q_2$ This system can be solved by the CRT, since the $Q_i$ are pairwise comaximal, since nonzero primes of a number ring are maximal. These congruences say $\alpha\in Q_2$ but not any other $Q$, since $0\equiv 1$ only in the unit ideal. Define $\beta = \prod_{\sigma \in G(L/K)} \sigma( \alpha) \in \ZZ_K$, and observe that $\beta\not\in Q_1$. If it were in $Q_1$, one $\sigma(\alpha) \in Q_1$ for some $\sigma \in G(L/K)$ which would force $\alpha = \sigma \sigma\inv ( \alpha) \in \sigma\inv (Q_1)$, which contradicts the choice of $\alpha$ since \( \sigma\inv(Q_1) \) is one of the $Q$s appearing in the second congruence. On the other hand we have $\beta\in Q_2$ since the identity is an element of $G(L/K)$ and $\alpha\in Q_2$ and $\beta$ is a multiple of $\alpha$. We know $\beta\in \ZZ_K$, so $\beta \in Q_2 \intersect \ZZ_K = P$. But this is fishy because $Q_1, Q_2$ both lie above $P$ and both must contain $P$: \begin{tikzcd} {Q_1} && {Q_2} \\ \\ & P \arrow[no head, from=1-1, to=3-2] \arrow[no head, from=1-3, to=3-2] \end{tikzcd} > [Link to Diagram](https://q.uiver.app/?q=WzAsMyxbMSwyLCJQIl0sWzAsMCwiUV8xIl0sWzIsMCwiUV8yIl0sWzEsMCwiIiwwLHsic3R5bGUiOnsiaGVhZCI6eyJuYW1lIjoibm9uZSJ9fX1dLFsyLDAsIiIsMix7InN0eWxlIjp7ImhlYWQiOnsibmFtZSI6Im5vbmUifX19XV0=) Since $\beta$ is in every ideal above $P$, in particular $\beta \in Q_1$ since $Q_1 \contains P$, and this is a contradiction. ::: :::{.proposition title="?"} If $Q, Q'$ both lie above $P$, then $e(Q/P) = e(Q'/P)$ and $f(Q/P) = f(Q'/P)$. ::: :::{.proof title="?"} Pick $\sigma \in G(L/K)$ with $\sigma(Q) = Q'$. Factor $P\ZZ_L = Q^{e(Q/P)} J$ where $J$ is a product of primes not equal to $Q$. Applying $\sigma$ yields $P\ZZ_L = Q'^{e(Q/P)} J'$ where $J'$ is a product of primes not equal to $Q'$. This factors $P\ZZ_L$ into primes, and by uniqueness of prime factorization this exponent has to be $e(Q'/P)$ by definition. For the $f$s, note that $\sigma$ induces a ring morphism between the residue fields: \[ \bar\sigma: \ZZ_K/Q &\to \ZZ_L/Q' \\ \alpha \mod Q &\mapsto \sigma(\alpha) \mod Q' .\] This is well-defined since $\sigma(Q) = Q'$, and is an isomorphism since the inverse comes from \( \sigma ^{-1}\). This will imply that the $f$s are the same: we're looking at the degree of these extensions over $\ZZ_K/P$. An element of $\ZZ_L/Q$ also belonging to $\ZZ_K/P$ (as a subfield) has the form $\alpha \mod Q$ where $\alpha\in \ZZ_K$, and under $\sigma$ this is sent to $\sigma(\alpha)\mod Q' = \alpha \mod Q'$, which is an element of the copy of $\ZZ_K/P\injects \ZZ_L/Q'$. So $\sigma$ identifies the copies of $\ZZ_K/P$ in either side, and \[ [\ZZ_L/Q : \ZZ_K/P] = [\ZZ_L/Q: \ZZ_K/P] .\] ::: :::{.theorem title="efg theorem for Galois extensions"} Let $L/K$ be a Galois extension of number fields and $P\in \spec \ZZ_K$ be nonzero. Let $e,f$ be the common $e, f$ for all $Q$ above $P$, and let $g$ be the number of distinct $Q$ above $P$. Then \[ efg = n \da [L:K] .\] ::: :::{.proof title="?"} Using the previous efg theorem, $\sum_{i=1}^g e_i f_i = n$, but by the previous proposition, all of the $e_i$ are the same and all of the $f_i$ are the same. ::: :::{.example title="?"} Let $L = \QQ(\zeta_5)$ over $K= \QQ$, which is Galois with $G(L/K) \cong (\ZZ/5)\units \cong \ZZ_4$. We know $\ZZ_L = \ZZ[\zeta_5]$, which actually happens for any $n$, and $\min_{\zeta_5}(x) = x^4 + x^3 + x^2 + x + 1$. Since $\ZZ_L$ is a monogenic, we can apply Dedekind-Kummer. - Factoring $2\ZZ_L = P$ yields a single factor, so $g=1$. Since $e$ is the common exponent, $e=1$, so $f=4$. If you factor $\min_{\zeta_5}$ mod 2, in order to get a single prime D-K says this must be irreducible and $f_i$ are the degrees of the irreducible factors. - $19\ZZ_L = P_1 P_2$, so $g=2, e=1$, and so $f=2$. - $11\ZZ_L = P_1 P_2 P_3 P_4$ yields $g=4, e=1, f=1$. - $5\ZZ_L = P_1^4$ so $g=1,e=4,f=1$. Can the combination $(e,f,g) = (2,2,1)$ occur? This would require a prime factoring as $P^2$, but the answer is know. In fact $e>1$ only happens for $5$ since this corresponds (for all applicable primes $p$, which is all primes since $\ZZ_L$ is monogenic here) to the polynomial having a repeated factor mod $p$. This would require $\discriminant(\min_{\zeta_5}(x)) \equiv 0 \mod p$, but you can show that $5$ is the only prime that divides this discriminant. So $5$ is the only prime that could ramify, so $e=2$ never happens. In general, for cyclotomic extensions, $p$ is always totally ramified and $e=p-1$. ::: ## Decomposition, Inertia, Frobenius :::{.definition title="Decomposition"} Let $L/K$ be a Galois extension of number fields and let $Q \in \spec \ZZ_L$ be nonzero. Then $Q$ lies above a unique prime $P \in \spec \ZZ_K$, where $P = Q \intersect \ZZ_K$. Define the **decomposition group of $Q$** is defined as \[ D(Q) = D(Q/P) = \ts{ \sigma \in G(L/K) \st \sigma(Q) = Q} .\] ::: :::{.definition title="?"} Better notation: define $\FF_Q \da \ZZ_L/Q$ and $\FF_P = \ZZ_K/P$, so $\FF_P \subseteq \FF_Q$. ::: :::{.remark} We know that the Galois group will take $Q$ to some prime above $P$, these are the ones that take $Q$ to itself. These are the automorphisms that make sense "modulo $Q$": there is a group morphism \[ \red_{Q/P}: D(G/P) &\to G(\FF_Q/\FF_P) \\ \sigma &\mapsto \bar\sigma .\] where we reduce $\sigma$ mod $Q$, so $\bar\sigma(\alpha \mod Q) \da \sigma(\alpha) \mod Q$. One can check - For $\sigma \in D(Q/P)$, $\bar\sigma$ is a well-defined automorphism of $\FF_Q$ fixing $\FF_P$, since the elements in $\FF_P$ are of the form $\alpha \mod Q$ where $\alpha\in \ZZ_K$ and $\sigma$ fixes $K$. This crucially uses that $\sigma$ fixes $Q$, otherwise it won't be well-defined. - $\red_{Q/P}$ defines a group morphism. We'll see later that this is in fact a surjective morphism. So all automorphisms in the Galois group $G(\FF_Q/\FF_P)$ are reductions mod $Q$ of automorphisms in the decomposition group, which are the only ones that make sense to reduce mod $Q$ anyway! :::